Soft Tissue Infection Flashcards

1
Q
A 45-year-old gardener with a 10-year history of poorly controlled type 2 diabetes mellitus comes to the emergency department because of excruciating pain and swelling of the left forearm 1 day after he scraped his left arm on a rosebush. Physical examination shows extreme tenderness, edema, and crepitus. Which of the following is the most appropriate management?
A) CT scan
B) Econazole
C) Hyperbaric oxygen
D) Immediate surgical debridement
A

D) Immediate surgical debridement

How well did you know this?
1
Not at all
2
3
4
5
Perfectly
2
Q

Necrotizing fasciitis infects _____ and spares _____

A

Necrotizing fasciitis is a severe soft-tissue infection affecting the skin, subcutaneous tissue, and fascia. It characteristically spares the underlying muscle.

How well did you know this?
1
Not at all
2
3
4
5
Perfectly
3
Q

Necrotizing fasciitis inciting event

A

The inciting event for necrotizing fasciitis is often trauma, even minor trauma and small puncture wounds, but hematogenous spread is also a recognized etiology

How well did you know this?
1
Not at all
2
3
4
5
Perfectly
4
Q

Most common diseases associated with necrotizing fasciitis

A

The vast majority of patients have some form of chronic debilitating disease that weakens the immune system. Diabetes mellitus appears to be the most common disease, but these may include substance abuse and renal failure. These patients are at higher risk for increased mortality

How well did you know this?
1
Not at all
2
3
4
5
Perfectly
5
Q

Type I necrotizing fasciitis

A

Two types of necrotizing fasciitis have been described. Type I are mixed aerobic and anaerobic infections, with facultative anaerobic bacteria and non-group A streptococci being present. This is the most common type and is present in about 75% of cases.

How well did you know this?
1
Not at all
2
3
4
5
Perfectly
6
Q

Type II necrotizing fasciitis

A

Type II infections are monomicrobic and are caused by group A Streptococcus species alone or in combination with staphylococcal species. For this reason, antibiotic coverage should be broad

How well did you know this?
1
Not at all
2
3
4
5
Perfectly
7
Q

Progression / stages of necrotizing fasciitis

A

The disease progresses quickly but does evolve through several stages. Initial symptoms include tenderness, erythema, edema, warm skin, and fever; however, symptoms may vary depending on patient characteristics. Initial lab findings may include leukocytosis, thrombocytopenia, and hyperkalemia, but these are variable. When critical skin ischemia occurs, blisters or bullae are formed. In the late stage, lesions turn black and necrotic and are anesthetic as the nerves become involved.

How well did you know this?
1
Not at all
2
3
4
5
Perfectly
8
Q

Mortality rate for necrotizing fasciitis, and patient population with increased mortality

A

Mortality rates for necrotizing fasciitis are reported to be 10 to 75% and are increased in patients with underlying immune compromise, delayed treatment, or involvement of the chest wall.

How well did you know this?
1
Not at all
2
3
4
5
Perfectly
9
Q

Hyperbaric oxygen and necrotizing fasciitis

A

Most authorities agree that hyperbaric oxygen is to be recommended for the treatment of necrotizing fasciitis as an adjunct, if facilities are available and there is no delay in surgical debridement. However, most studies regarding the efficacy of hyperbaric oxygen are anecdotal with a distinct lack of properly designed prospective randomized controlled trials.

How well did you know this?
1
Not at all
2
3
4
5
Perfectly
10
Q

A 24-year-old man comes to the emergency department 6 hours after sustaining an open distal radius fracture and loss of soft tissue while he was working on a farm. History includes diabetes mellitus. On physical examination, distal pulses and sensation are intact. Debridement and repair with open reduction and internal fixation are planned. Which of the following factors places this patient at greatest risk for infection postoperatively?
A) Gustilo fracture classification
B) History of diabetes mellitus
C) Method of fracture fixation
D) Period of time from injury to initial debridement
E) Type of contamination

A

E) Type of contamination

How well did you know this?
1
Not at all
2
3
4
5
Perfectly
11
Q

Correlation of Gustilo-Anderson fracture classification and infection - upper extremity

A

The Gustilo–Anderson fracture classification has been found to have some correlation with open fracture infection rates, but this relationship is not as strong in the distal radius as it is in long bones of the lower extremity.

How well did you know this?
1
Not at all
2
3
4
5
Perfectly
12
Q

Systematic illness and correlation with infection in distal radius fractures

A

A history of systemic illness, such as diabetes mellitus, and the type of fracture fixation seem to have a modest correlation to postoperative infections in distal radius fractures, but the findings are generally not statistically significant.

How well did you know this?
1
Not at all
2
3
4
5
Perfectly
13
Q

Schedule of debridement vs infection rate

A

The timing of the initial debridement, as long as it occurs within the first 24 hours of injury, does not significantly affect infection rate. In contrast, there is evidence to suggest that performing multiple serial debridements in significantly contaminated wounds is of benefit in preventing postoperative infection.

How well did you know this?
1
Not at all
2
3
4
5
Perfectly
14
Q
A 45-year-old woman comes to the emergency department because of redness at the site of injury 24 hours after being bitten by a cat. Temperature is 98.6 °F (37.0 °C), heart rate is 77 bpm, respirations are 16/min, and blood pressure is 125/82 mmHg. Physical examination shows mild erythema and edema around the puncture marks over the volar second phalanx of the long finger of the left hand. No purulent material is expressed. Resting position of the digit is normal, and there is no pain with passive extension. Assuming that the patient has no allergies to medications, which of the following is the most appropriate antimicrobial therapy?
A ) Amoxicillin-clavulanate
B ) Cefazolin
C ) Cephalexin
D ) Erythromycin
E ) Vancomycin and gentamicin
A

A ) Amoxicillin-clavulanate

How well did you know this?
1
Not at all
2
3
4
5
Perfectly
15
Q

Common organisms involved in cat bite infections

A

Cat bites usually result in puncture wounds because of their long, slender, sharp teeth. It has been reported that as many as 80%of cat bites become infected. Pasteurella species are the most common, occurring in as many as 75% of cat bite infections. Other types of bacteria can also be involved as well, including Streptococci, Staphylococci, and anaerobes.

How well did you know this?
1
Not at all
2
3
4
5
Perfectly
16
Q

Pasteurella species are susceptible to _____ and resistant to ______

A

Pasteurella species are susceptible to penicillin, ampicillin, second-and third-generation cephalosporins, doxycycline, trimethoprim-sulfamethoxazole, fluoroquinolones, clarithromycin, and azithromycin, but are resistant to cephalexin, dicloxacillin, erythromycin, and clindamycin.

How well did you know this?
1
Not at all
2
3
4
5
Perfectly
17
Q

Appropriate choices for prophylaxis and treatment of cat bite wounds

A

Appropriate choices for prophylaxis and treatment of cat bite wounds would include amoxicillin and clavulanate or a combination of penicillin plus cephalexin. For those patients who have an allergy to penicillin, moxifloxacin or combination therapy with ciprofloxacin and clindamycin are appropriate choices. Azithromycin may also be effective for the penicillin-allergic patient, but it has less activity against anaerobes.

Coverage for community-acquired MRSA (CA-MRSA) infection is not recommended because oral colonization of the human and animal mouth with CA-MRSA is unlikely.

How well did you know this?
1
Not at all
2
3
4
5
Perfectly
18
Q

A 40-year-old woman is undergoing chemotherapy for metastatic lung cancer. During administration of her first dose of doxorubicin, she reports pain at the site of injection. The following day, physical examination shows the hand to be swollen and an ulcer measuring 2x3 cm is seen over the dorsum surrounded by an area of ischemia. Which of the following is the most appropriate immediate treatment?
A ) Administration of hyperbaric oxygen
B ) Applicationof cold packs
C ) Application of hot packs
D ) Application of topical dimethyl sulfoxide
E ) Immediate surgical excision and autografting

A

D ) Application of topical dimethyl sulfoxide

In the case of doxorubicin extravasation injury, cold compresses may exacerbate the complication by venous constriction, which localizes the drug, whereas hot packs may cause vasodilatation with further extravasation.

How well did you know this?
1
Not at all
2
3
4
5
Perfectly
19
Q

The specific treatment of extravasation injury is dependent on:

A

The specific treatment of an extravasation injury is dependent on the drug infused.

How well did you know this?
1
Not at all
2
3
4
5
Perfectly
20
Q

Treatment of extravasation of anthracyclines

A

Application of topical dimethyl sulfoxide has been advocated for the treatment of extravasation of anthracyclines and is supported by several studies.

How well did you know this?
1
Not at all
2
3
4
5
Perfectly
21
Q

Extravasation of surgical excision

A

Early excision for extravasation injury is rarely performed without evidence of at least ulceration, and the main indication would then be for pain control. In the scenario described, an option would be early surgical debridement and delayed closure of the wound; however, some of the surrounding skin may heal. Thus, a conservative initial approach with later excision after 2 to 3 weeks is recommended since this will give a better cosmetic result.

How well did you know this?
1
Not at all
2
3
4
5
Perfectly
22
Q
A 43-year-old man who is HIV positive comes to the emergency department because of a 1-month history of gradually progressive swelling and pain in the tip of the right index finger. Examination shows significant swelling, erythema, and small vesicles on the fingertip and perionychium. A few nodular, reddish brown lesions are noted on the chest and forearms. Which of the following is the most appropriate management? 
A ) Administration of acyclovir
 B ) Administration of cefuroxime 
C ) Administration of ketoconazole cream
D ) Excision of proximal nail plate
E ) Incision and drainage
A

A ) Administration of acyclovir

How well did you know this?
1
Not at all
2
3
4
5
Perfectly
23
Q

Diagnosis of HSV infection of the hand

A

Herpes simplex virus (HSV) infection of the hand is clinically diagnosed when characteristic multiple vesicular lesions on an erythematous base are present.

The staining of scrapings from the base of the lesions with a Tzanck preparation for a Papanicolaou stain demonstrates giant cells or intranuclear inclusions of HSV infection.

How well did you know this?
1
Not at all
2
3
4
5
Perfectly
24
Q

Duration of HSV infection of the hand

A

HSV hand infection generally resolves within 3 weeks.

How well did you know this?
1
Not at all
2
3
4
5
Perfectly
25
Q

In case of prolonged HSV infection of the hand

A

HSV hand infection generally resolves within 3 weeks. If symptoms last longer than 3 weeks, suggestion of immunosuppression or AIDS should be raised.

How well did you know this?
1
Not at all
2
3
4
5
Perfectly
26
Q

Treatment of HSV infection of the hand

A

Treatment with an antiviral such as acyclovir is recommended for HSV infection of the hand

How well did you know this?
1
Not at all
2
3
4
5
Perfectly
27
Q

Cause of acute paronychia

A

Acute paronychia is most commonly caused by bacterial inoculation, and the most common agent is Staphylococcus aureus

How well did you know this?
1
Not at all
2
3
4
5
Perfectly
28
Q

Cause of chronic paronychia

A

Chronic bacterial infections of the subungual space are usually secondary infections of a preexisting fungal infection.

The fungal organismsare the primary cause of chronic paronychia. Candida is the most common infecting organism. Most often affecting women, this low-grade smoldering infection causes thickening and fibrosis.

How well did you know this?
1
Not at all
2
3
4
5
Perfectly
29
Q

Treatment of chronic paronychia

A

Topical and systemic antifungal agents have been used for treatmentof minimal-to-moderate fungal infections. For extensive infection, the nail plate is removed and topical antifungals are used.

How well did you know this?
1
Not at all
2
3
4
5
Perfectly
30
Q
A 60-year-old woman with type 1 diabetes mellitus has a 1-cm puncture wound to the lower leg sustained 2 days ago. Physical examination shows erythema of the surrounding tissue, tenderness of the entire calf, and crepitus. Which of the following is the most appropriate initial management of the wound? 
A ) Hyperbaric oxygen therapy 
B ) Silver sulfadiazine dressing
C ) Surgical debridement
D ) Unna boot compression dressing
E ) Vacuum-assisted closure therapy
A

C ) Surgical debridement

Necrotizing fasciitis must be suspected in any patient with a compromised immune system, even when he or she has a relatively small surfacewound.

How well did you know this?
1
Not at all
2
3
4
5
Perfectly
31
Q

A 25-year-old man who is a soldier from Afghanistan is evaluated because of deep frostbite of the right hand after being in the field for 36 hours. Rewarming is performed in the field. He is transferred to a hospital for further evaluation. Physical examination shows hemorrhagic blisters and eschar formation on the hand, erythema of the surrounding area, and streaking up the forearm. Which of the following is the most appropriate next step in management?
A ) Administration of dextran
B ) Administration of penicillin
C ) Hyperbaric oxygen therapy
D ) Intra-arterial injection of reserpine
E ) Observation

A

B ) Administration of penicillin

How well did you know this?
1
Not at all
2
3
4
5
Perfectly
32
Q

Field management of frostbite

A

Field management for frostbite includes rapid rewarming of the affected area with circulating water at 104 to 107.6 °F (40 to 42 °C) for a period of 15 to 30 minutes, protection from mechanical trauma, and appropriate analgesia.

How well did you know this?
1
Not at all
2
3
4
5
Perfectly
33
Q

Following initial field management of frostbite

A

Next steps in management of frostbite following initial field management include elevation, antitetanus prophylaxis, debridement of clear blisters, leaving hemorrhagic blisters intact, and application of aloe vera. Penicillin should be administered for cellulitis.

How well did you know this?
1
Not at all
2
3
4
5
Perfectly
34
Q

Adjuvant therapies for frostbite for which there is equivocal/scant data

A

Adjuvant therapies can include anticoagulation, thrombolytics, hyperbaric oxygen, and sympathetic blockade; however, data to support these therapies are scant and equivocal at best.

How well did you know this?
1
Not at all
2
3
4
5
Perfectly
35
Q

Timing for definitive surgical amputation after frostbite

A

Definitive surgical amputation following frostbite should be delayed for at least 3 weeks to allow for tissues to demarcate, in terms of viability

How well did you know this?
1
Not at all
2
3
4
5
Perfectly
36
Q

A 55-year-old man comes to the emergency department because of acute onset of pain, swelling, and erythema of the right groin, lower abdominal wall, and right scrotum. He has type 1 diabetes mellitus which is controlled with insulin. He has smoked one pack of cigarettes daily for the past 30 years. He weighs 145 kg (320 lb); BMI is 35 kg/m2. Temperature is 38.3°C (101°F), pulse is 136 bpm, and blood pressure is 90/40 mmHg. In addition to admission to the hospital and administration of pain medication, which of the following is the most appropriate sequence in management?
A ) Intravenous antibiotics, hyperbaric oxygen, intravenous hydration
B ) Intravenous hydration, immediate surgical debridement, right orchiectomy
C ) Intravenous hydration, intravenous broad-spectrum antibiotics, CT of abdomen
D ) Intravenous hydration, intravenous broad-spectrum antibiotics, immediate surgical debridement
E ) Observation, intravenous antibiotics

A

D ) Intravenous hydration, intravenous broad-spectrum antibiotics, immediate surgical debridement

Orchiectomy is rarely needed, as testicles have their own blood supply and are protected by external spermatic fascia.

How well did you know this?
1
Not at all
2
3
4
5
Perfectly
37
Q

Predisposing factors for Fournier disease

A

Diabetes mellitus, alcoholism, heavy smoking, leukemia, and AIDS can predispose to Fournier disease.

How well did you know this?
1
Not at all
2
3
4
5
Perfectly
38
Q

Common sources of infection for Fournier disease

A

Common sources of Fournier disease include urogenital disease, trauma, or recent manipulation (iatrogenic trauma, ie, endoscopic procedures)

How well did you know this?
1
Not at all
2
3
4
5
Perfectly
39
Q

Antibiotics for Fournier disease should include

A

Key steps in treatment of Fournier disease include early diagnosis, intravenous hydration, and broad-spectrum antibiotics (anaerobic coverage for Clostridium perfringensshould continue as it is difficult to culture this).

How well did you know this?
1
Not at all
2
3
4
5
Perfectly
40
Q

Mortality rate from Fournier gangrene

A

Mortality rate for Fournier gangrene is quoted to be 7% to 75%, with the higher rate in patients with diabetes, alcoholism, and in cases of delayed diagnosis

How well did you know this?
1
Not at all
2
3
4
5
Perfectly
41
Q
A 56-year-old woman develops a necrotizing infection of the abdominal wall (shown). Appropriate antibiotics are started and devitalized tissue is debrided. Which of the following interventions is most likely to decrease mortality?
A ) High-dose corticosteroids
B ) Hyperbaric oxygen treatment
C ) Intravenous gamma globulin
D ) Topical papain-urea ointment
E ) Vacuum-assisted closure device
A

B ) Hyperbaric oxygen treatment

How well did you know this?
1
Not at all
2
3
4
5
Perfectly
42
Q

Some studies have shown that mortality rate of necrotizing infections can be reduced with:

A

Necrotizing soft-tissue infections are rapidly progressive, polymicrobial infections that require prompt diagnosis, debridement of devitalized tissue, and administration of appropriate antibiotics. Some studies have shown that mortality rates in necrotizing infections can be reduced with the addition of hyperbaric oxygen treatment.

How well did you know this?
1
Not at all
2
3
4
5
Perfectly
43
Q

A 23-year-old man is brought to the emergency department one hour after sustaining a snake bite to the lower right leg while hiking. The description of the snake is consistent with that of a copperhead. The patient does not recall his immunization status. Vital signs are stable. Physical examination shows mild edema around the bite. No neurologic deficits are noted. Coagulation and routine laboratory studies are ordered. Intravenous fluids are started. Which of the following is the most appropriate next step in management?
A ) Administration of equine antivenin
B ) Administration of tetanus toxoid
C ) Debridement of the wound and initiation of suction therapy
D ) Elevation and application of a tourniquet
E ) Prophylactic fasciotomy

A

B ) Administration of tetanus toxoid

How well did you know this?
1
Not at all
2
3
4
5
Perfectly
44
Q

Percent of snake bites that are dry bites

A

10% to 50% of snake bites have been reported as dry bites

How well did you know this?
1
Not at all
2
3
4
5
Perfectly
45
Q

First aid after a snake bite

A

Acute first aid interventions after snake bite include avoiding excessive activity of the affected site, immobilization in a neutral position, and expeditious transportation to a hospital.

How well did you know this?
1
Not at all
2
3
4
5
Perfectly
46
Q

Potential complications of antivenin

A

Anaphylaxis and serum sickness are potential significant complications of equine antivenin.

How well did you know this?
1
Not at all
2
3
4
5
Perfectly
47
Q

Indication for antivenin administration

A

The general indication for antivenin administration is progressive venom injury. This is defined as worsening local injury (eg,swelling, pain, or ecchymosis), the onset of clinical coagulopathy, or the development of systemic effects including hypotension or changes in mental status. The patient should be monitored closely because the clinical presentation can change rapidly. Antivenin therapy should be institutedif signs or symptoms of envenomation develop or progress.

How well did you know this?
1
Not at all
2
3
4
5
Perfectly
48
Q

Snake with the most potent venom

A

Rattlesnake bites deliver the most potent venom and are responsible for the majority of fatalities from snake bites;

How well did you know this?
1
Not at all
2
3
4
5
Perfectly
49
Q

Debridement of snake bite site

A

Debridement of the bite site and suction therapy have not been shown to be beneficial in reducing the effects of envenomation and can cause additional necrosis to the tissue under the suction cup.

How well did you know this?
1
Not at all
2
3
4
5
Perfectly
50
Q

Suction therapy of a snake bite site

A

Debridement of the bite site and suction therapy have not been shown to be beneficial in reducing the effects of envenomation and can cause additional necrosis to the tissue under the suction cup.

How well did you know this?
1
Not at all
2
3
4
5
Perfectly
51
Q

Arterial tourniquet of a snake bite site

A

Arterial tourniquets are contraindicated because they may worsen tissue ischemia and necrosis

How well did you know this?
1
Not at all
2
3
4
5
Perfectly
52
Q

Concern for compartment syndrome after snake bite

A

Fasciotomy should be performed only for clinical signs and symptoms of compartment syndrome. Compartment syndrome and infection from extremity bites are extremely rare.

But watch for it!

How well did you know this?
1
Not at all
2
3
4
5
Perfectly
53
Q

A 57-year-old man with type 2 diabetes mellitus comes to the emergency department because of redness and swelling of the dorsum of his right hand five hours after being bitten by an insect. Intravenous first-generation cephalosporin is started. Three hours later, the area of redness has extended to his shoulder and chest; temperature is 103.8°F (39.9°C), pulse is 110 bpm, and blood pressure is 100/55 mmHg. Which of the following is the most appropriate management?
A ) Blood cultures and empiric addition of anti fungal antibiotics
B ) Elevation and ice compression to his extremity
C ) Fasciotomy of compartments
D ) Increase in dosage of intravenous antibiotics
E ) Operative incision, drainage, and debridement of affected tissues

A

E ) Operative incision, drainage, and debridement of affected tissues

How well did you know this?
1
Not at all
2
3
4
5
Perfectly
54
Q

A 15-year-old girl is brought to the emergency department 24 hours after being bitten on the left index finger pulp pad while playing with her friend’s cat. The patient received a tetanus immunization three years ago. Temperature is 37°C (98.6°F). Physical examination shows mild erythema surrounding a fluctuant area beneath a sealed puncture wound. No motor or sensory deficits are noted. Radiographs show no abnormalities. Leukocyte count is 12,800/mm3. In addition to adequate drainage of the wound, which of the following is the most appropriate next step in management?
A ) Administration of amoxicillin and clavulanate
B ) Administration of ciprofloxacin
C ) Administration of rabies toxoid
D ) Administration of tetanus immune globulin
E ) Observation

A

A ) Administration of amoxicillin and clavulanate

How well did you know this?
1
Not at all
2
3
4
5
Perfectly
55
Q

Microbiology/treatment of cat and dog bites

A

Cat and dog bite infections have a complex microbiologic mix, with Pasteurellaas the most frequent isolate. Common aerobes isolated include streptococci, staphylococci, Moraxella, and Neisseria; common anaerobes include Fusobacterium, Bacteroides, Porphyromonas, and Prevotella.

How well did you know this?
1
Not at all
2
3
4
5
Perfectly
56
Q

Antibiotic treatment for cat/dog bites

A

Patients are most often treated with a combination of a β-lactam antibiotic and a β-lactamase inhibitor, such as amoxicillin and clavulanic acid.

Alternative treatments include clindamycin plus fluoroquinolone for adults and clindamycin and trimethoprim/sulfamethoxazole for children.

How well did you know this?
1
Not at all
2
3
4
5
Perfectly
57
Q

Surgical management of cat vs dog bites

A

In general, cat bites tend to be small puncture wounds that seal almost immediately; thus, wounds should be opened sufficiently to be cleaned and allowed to drain. Dog bite wounds tend to be longitudinal lacerations that are more likely to stay open by themselves. However, canine lacerations, as compared to cat bite punctures, are more likely to result in direct structural damage to nerves, tendons, vessels, and joint structures.

How well did you know this?
1
Not at all
2
3
4
5
Perfectly
58
Q

Indication for administration of tetanus immune globulin

A

Administration of tetanus immune globulin is recommended only for patients with tetanus-prone wounds who have never completed a primary immunization series.

How well did you know this?
1
Not at all
2
3
4
5
Perfectly
59
Q

Tetanus prophylaxus for patient with acute soft tissue injury and no previous immunization

A

If a patient with an acute soft-tissue injury has not been immunized previously, a tetanus toxoid booster is required. The patient must complete the series.

How well did you know this?
1
Not at all
2
3
4
5
Perfectly
60
Q

Booster doses for tetanus for previously immunized patient after acute soft tissue injury

A

If the patient has been immunized previously, a booster dose is given if the last dose was more than five years previously (for a tetanus-prone wound) or more than 10 years previously (for a nontetanus-prone wound).

How well did you know this?
1
Not at all
2
3
4
5
Perfectly
61
Q

Management of patients with acute soft tissue injury and contraindication to tetanus toxoid

A

Patients with a contraindication to tetanus toxoid must be managed with tetanus immune globulin alone.

How well did you know this?
1
Not at all
2
3
4
5
Perfectly
62
Q
A 53-year-old man returns to the emergency department because he has redness, swelling, and severe pain in the left upper extremity two days after he sustained a laceration to the left hand. Current temperature is 102°F (38.9°C) and heart rate is 126/min. Current examination shows crepitus extending into the left forearm. Radiograph shows some gas in the softtissue. Which of the following is the most likely causative organism?
A ) Eikenella corrodens
B ) Pasteurella multocida
C ) Pseudomonas aeruginosa
D ) Staphylococcus epidermidis
E ) Streptococcus pyogenes
A

E ) Streptococcus pyogenes

How well did you know this?
1
Not at all
2
3
4
5
Perfectly
63
Q

Progressive clinical presentation of necrotizing fasciitis

A

Symptoms usually begin with localized erythema and swelling and may mimic cellulitisin the early stages. Severe pain, crepitus, and systemic toxicity can provide clues to the diagnosis. Radiographs may show air in the soft tissues, and patients may exhibit grayish, watery discharge (dishwater pus).

How well did you know this?
1
Not at all
2
3
4
5
Perfectly
64
Q

Eikenella corrodens

A

Eikenella corrodensis an anaerobic organism present in human oral flora and has been associated with human bite wounds.

How well did you know this?
1
Not at all
2
3
4
5
Perfectly
65
Q

Pasteurella multocida

A

Pasteurella multocidais a gram-negative anaerobic bacterium most commonly associated with cat bite infections.

How well did you know this?
1
Not at all
2
3
4
5
Perfectly
66
Q

Staphylococcus epidermidis

A

Staphylococcus epidermidisis a gram-positive coccus present on the skin. It has been associated with implant infections.

How well did you know this?
1
Not at all
2
3
4
5
Perfectly
67
Q

A 2-year-old boy is brought to the emergency department because he has had lethargy, fever, and a rash over the extremities for the past 10 hours. Temperature is 39.9EC (103.8EF). Physical examination shows petechiae over the trunk and arms. Over thenext three hours, the rash coalesces to hemorrhagic bullae, and the diagnosis of purpura fulminans is confirmed. Each of the following management interventions is appropriate EXCEPT
(A)administration of activated protein C
(B)broad-spectrum antibiotic therapy
(C)early wound debridement and amputation of ischemic digits
(D)fasciotomy of extremities
(E)fluid resuscitation with inotropic support

A

(C)early wound debridement and amputation of ischemic digits

How well did you know this?
1
Not at all
2
3
4
5
Perfectly
68
Q

Purpura fulminans

A

Purpura fulminans is a frequently fatal, rapidly evolving syndrome of septic shock and hemorrhagic bullae, which can result in massive desquamation.

How well did you know this?
1
Not at all
2
3
4
5
Perfectly
69
Q

Management of purpura fulminans

A

Management includes prompt recognition of the infection (which is usually caused by Neisseria meningitidis), initiation of broad-spectrum antibiotic therapy, mechanical ventilation, and aggressive fluid resuscitation with inotropic support. Disseminated intravascular coagulopathy (DIC) develops, and patients seem to benefit from replacement of activated protein C.

How well did you know this?
1
Not at all
2
3
4
5
Perfectly
70
Q

Purpura fulminans is most often caused by

A

usually caused by Neisseria meningitidis

How well did you know this?
1
Not at all
2
3
4
5
Perfectly
71
Q

Improving limb salvage in purpura fulminans

A

need for early fasciotomy to improve limb salvage. It is difficult to determine tissue viability during the resuscitation period; therefore, debridement, coverage, and amputation are delayed until demarcation has occurred

How well did you know this?
1
Not at all
2
3
4
5
Perfectly
72
Q
A 27-year-old woman with a history ofintravenous drug use has a persistent abscess and cellulitis of the dorsal aspect of the left hand. A photograph of the hand is shown. Three days ago, incision and drainage were performed using local anesthesia in the emergency department, and the patientwas then admitted to the hospital for intravenous administration of piperacillin with tazobactam. Results of culture of thwound tissue are pending. Current physical examination shows that the cellulitis has not resolved satisfactorily. Which of the following is the most likely causative organism of this infection?
(A)Bacteroides fragilis
(B)Escherichia coli
(C)Group B streptococcus
(D)Pseudomonas aeruginosa
(E)Staphylococcus aureus
A

(E)Staphylococcus aureus

The infection described is most likely caused by a subset of Staphylococcus aureus, which is methicillin-resistant

How well did you know this?
1
Not at all
2
3
4
5
Perfectly
73
Q
A 38-year-old man undergoes extensive debridement of skin, subcutaneous tissue, and fascia on the right chest wall (shown) for progressive Type II monomicrobial necrotizing fasciitis. Results of culture are pending. In combination with penicillin, which of the following antibiotics is the most appropriate initial therapy?
(A)Ciprofloxacin
(B)Clindamycin
(C)Gentamicin
(D)Metronidazole
(E)Vancomycin
A

(B)Clindamycin

74
Q

Type II necrotizing fasciitis

A

Type II is a monomicrobial infection that can occur in healthy individuals in any age group. Often, the individual has a history of blunt trauma, penetrating injuries or lacerations (often minor), surgical procedures, childbirth, or burns

75
Q

Treatment of Type II necrotizing fasciitis

A

In Type II fasciitis, the most common causative organisms are group A streptococci. Clindamycin and penicillin in combination are recommended. Clindamycin has been shown to suppress toxin production, whereas metronidazole has not. Clindamycin also facilitates phagocytosis of Streptococcus pyogenes. Penicillin is added because of increasing resistance of group A streptococci to clindamycin.Infection with Staphylococcus aureusis a less common cause of Type II necrotizing fasciitis. Oxacillin could be used to treat susceptible S. aureusinfection, whereas vancomycin is appropriate therapy for methicillin-resistant S. aureus(MRSA)

76
Q

Type I Necrotizing fasciitis

A

Type I is a polymicrobial infection caused by aerobic and anaerobic bacteria occurring primarily in patients who are immunocompromised or have certain chronic diseases like diabetes.

77
Q

Treatment of Type I Necrotizing fasciitis

A

Type I necrotizing fasciitis requires a multiple antibiotic regimen to cover aerobic and anaerobic bacteria. Therefore, broad-spectrum coverage with ampicillin-sulbactam, clindamycin, ciprofloxacin, or gentamicin is recommended.

78
Q

A 45-year-old woman is referred to the office because she has had chronic infection, drainage, and pain of the left armpit for the past 10 years. The contralateral axilla is similarly affected. Medical history includes multiple courses of antibiotic therapy and intermittent drainage procedures. Physical examination shows a 10 x 15-cm area of involvement. Which ofthe following is the most effective management?
(A)Oral administration of an antibiotic
(B)Intravenous administration of an antibiotic
(C)Injection of botulinum toxin A
(D)Incision and drainage of the region
(E)Complete excision of the scarred area

A

(E)Complete excision of the scarred area

79
Q

Hidradenitis suppurativa

A

Hidradenitis suppurativa is an inflammatory disease of the apocrine glands and follicular epithelium, presenting most commonly as deep recurrent infections or chronic sinus tracts inhair-bearing regions of the skin. It is most commonly seen in the axillary region in young women, but the groin and perineum may also be affected

80
Q

Treatment of hidradenitis suppurativa

A

After the diagnosis has been confirmed, a short course of treatment with local drainage and suppressive antibiotics is appropriate. However, once the disease process is established, the only appropriate therapy is complete excision of the scarred area.

The mainstay of treatment for larger areas is complete excision of axillary skin followed by split-thickness skin grafting. Negative pressure dressings are helpful to stabilize the graft. Although numerous local fasciocutaneous pedicled and free flaps have been described, they increase the risk of donor site morbidity and are generally not necessary. Healing by secondary intention has also been described. However, it is preferable to undertake excision of one axilla at a time so the patient is not disabled in the perioperative period.

81
Q

Botulinum toxin and hidradenitis suppurativa

A

The successful treatment of chronic hidradenitis with injection of botulinum toxin A has been reported. Sweat glands have a sympathetic innervation. Botulinum toxin works by blocking the release of acetylcholine from nerve endings, thus decreasing sweat production. However, this is expensive and represents a temporary treatment of a chronic problem

82
Q
A 26-year-old man comes to the office because he has worsening pain and erythema in the thumb and small finger of the dominant right hand two days after he punctured the thumb with a wood splinter. The most appropriate initial step is exploration of the thumb, small finger, and which of the following?
(A)Dorsum of the hand
(B)Midpalmar space
(C)Parona space
(D)Posterior adductor space
(E)Ring finger
A

(C)Parona space

83
Q

The parona space

A

The radial and ulnar bursas are connected through the Parona space,which lies between the pronator quadratus fascia and the flexor digitorum profundus tendon sheath.

The Parona space can be explored by performing an extended carpal tunnel release

84
Q

The flexor tendon sheath of the small finger

A

The flexor tendon sheath of the small finger often connects with the ulna bursa, which extends proximal to the transverse carpal ligament.

85
Q

“Horseshoe abscess”

A

This abscess results when an infection starting in the thumb or small finger progresses proximal through the wrist and then into the opposite flexor tendon sheath through the Parona space.

86
Q

Thenar space

A

Another potential space in the hand is the thenar space, which is bordered ulnarly by the vertical septum between the flexor sheath and metacarpal of the long finger, dorsally by the fascia of the adductor pollicis, and radially by the thenar muscle fascia. It should be noted that this space does not include the thenar muscles.

87
Q

Midpalmar space

A

The midpalmar space is bordered radially by the vertical septum between the flexor sheath and metacarpal of the long finger,dorsally by the fascia over the interossei of the third and fourth web spaces, ulnarly by the fascia over the hypothenar muscles, and volarly by the flexor tendons

88
Q

Posterior adductor space

A

The posterior adductor space is defined as the space dorsal to the adductor pollicis andvolar to the first dorsal interosseous

89
Q

Posterior interosseous space

A

The dorsum of the hand contains the posterior interosseous space, which is dorsal to the first dorsal interosseous.

90
Q

A 19-year-old man has a seven-day history of swelling and pain on the volar surface of the left index finger. He has no history of trauma to the finger. Gram’s stain of fluid aspirated from the joint shows gram-negative diplococci.
Treat with:

A

Disseminated gonococcal infection is the most common cause of acute infectious arthritis in sexually active adults. A history or evidence of trauma are lacking. Fluid aspiration with Gram staining for gram-negative diplococci allows definitive diagnosis. Hospitalization and intravenous administration of a third-generation cephalosporin is recommended.

91
Q

A 45-year-old man has a two-day history of pain and swelling of the proximal interphalangeal joint of the right index finger. Physical examination shows pain on passive motion of the affected joint and erythema and tenderness extending into the right hand. Gram’s stain of fluid aspirated from the joint shows gram-positive cocci.

A

The two most common organisms that cause hand infections are Staphylococcus aureusand B-hemolytic streptococci. Minor staphylococcal and streptococcal infections are treated with first-generation cephalosporins. More significant infections of the interphalangeal joint should be performed through a midaxial incision

92
Q

General overview of septic arthritis

A

Septic arthritis may result from extension of an adjacent subcutaneous abscess or by intra-articular contamination caused by a laceration or puncture wound. The joint is a poorly vascularized potential space, favoring colonization. Early diagnosis and drainage are crucial to treatment, as a joint infection can progress rapidly to destruction of articular cartilage.

93
Q

Most common organisms that cause hand infection

A

The two most common organisms that cause hand infections are Staphylococcus aureusand B-hemolytic streptococci.

Neisseria gonorrhoeae usually manifests as a primary venereal infection. However, it can disseminate and sometimes present as a secondary hand infection, which is often confused with a purulent tenosynovitis or arthritis.

94
Q

Importance of differentiating neisseria hand infection from pyogenic infection

A

It is important to distinguish gonococcal from pyogenic infection because, unlike a pyogenic infection, a gonococcal infection does not usually destroy tendon or articular cartilage. Therefore, incision, drainage, and debridement are unnecessary and should be avoided.

95
Q

Sweet syndrome

A

Manifestations of Sweet syndrome can masquerade as acute hand infections. Sweet syndrome, originally described as an acute febrile neutrophilic dermatosis, belongs to a class of skin lesions that histologically have intense epidermal and/or dermal inflammatory infiltrate of neutrophils without evidence of infection or vasculitis. The lesions can erupt at sites of minor trauma. The clinical picture is consistent with infection initially. The unresponsiveness of these lesions to antimicrobial therapy and the lack of associated cellulitis is a clue to the diagnosis. The treatment of choice involves a tapering dose of corticosteroids.

96
Q

Herpetic whitlow

A

Herpetic whitlows are often confused with paronychia or felon and are treated mistakenly as such. Initial signs include intense pain and erythema of the fingertip, followed by edema and tenderness. A Tzanck smear of vesicular fluid may show multinucleated giant cells. Primary herpes simplex infections typically resolve without treatment within three weeks. Incision and drainage of herpetic whitlow is contraindicated because surgical treatment converts a closed wound to open and may result in a secondary bacterial infection or viral superinfection. To date, there have been no controlled studies that assess the efficacy of acyclovir for the treatment of herpetic whitlow, but case reports suggest that it both suppresses and decreases the length and severity of recurrent infections when taken orally.

97
Q
Which of the following is the most common causative organism of infectious folliculitis?
(A) Peptostreptococcus anaerobius
(B) Staphylococcus aureus
(C) Staphylococcus epidermidis
(D) Streptococcus milleri
(E) Streptococcus pyogenes
A

(B) Staphylococcus aureus

98
Q

Most common form of infectious folliculitis

A

Staphylococcal folliculitis is the most common form of infectious folliculitis. One or more pustules may appear, usually without fever or other systemic symptoms, on any body surface. Staphylococcal folliculitis may occur because of injury, abrasion, or nearby surgical wounds or draining abscesses.

99
Q

Streptococci soft tissue infections

A

The streptococci are secondary invaders of traumatic skin lesions and cause impetigo, erysipelas, cellulitis, and lymphangitis.

100
Q

Staphylococcus aureus soft tissue infections

A

S. aureus invades skin and causes impetigo, folliculitis, cellulitis, and furuncles. Elaboration of toxins by S. aureus causes the lesions of bullous impetigo and staphylococcal scalded skin syndrome

101
Q

A 9-year-old boy is brought to the emergency department because he has nausea and vomiting as well as pain in the left hand one hour after he sustained a snakebite during a camping trip. Physical examination shows fang marks on the left thumb and swelling of the distal aspect of the forearm. Sensation is intact and no ecchymosis is noted. Which of the following is the most appropriate management?
(A) Elevation of the extremity and application of a tourniquet
(B) Fluid resuscitation with normal saline
(C) Administration of antivenin after skin testing with dilute horse serum
(D) Immediate cryotherapy to the affected area of the hand
(E) Measurement of compartment pressures and subsequent fasciotomy

A

(C) Administration of antivenin after skin testing with dilute horse serum

In this clinical scenario of a snakebite to the upper extremity, the patient meets criteria for moderate envenomation, which can also yieldorthostatic changes and mild coagulation parameter changes. G

102
Q

Criteria for minimal envenomation

A

Criteria for minimal envenomation include fang marks, local swelling or pain, and no systemic reaction; therapy includes delivery of up to five vials of antivenin.

103
Q

Criteria for severe envenomation

A

Severe envenomation can produce subcutaneous ecchymosis, marked swelling of the extremity, coagulopathy, shock, and compartment syndrome; therapy includes delivery of 20 or more vials of antivenin (without delaying for skin testing), resuscitation with correction of acidosis and coagulopathy, and fasciotomy for compartment pressures greater than 30 mmHg or worsening findings on neurologic examination

104
Q

Criteria for moderate envenomation

A

Moderate envenomation can produce nausea, vomiting, swelling of the extremity; it can also yield orthostatic changes and mild coagulation parameter changes.

105
Q

Resuscitation after envenomation

A

Resuscitation should be performed with lactated Ringer’s solution. Normal saline is contraindicated because this solution will exacerbate metabolic acidosis due to its high chloride load (154 mEq/l).

106
Q

A 65-year-old woman comes to the office because she has had recurrent swelling of the right wrist over the past six months. She has also had some associated numbness of the index and long fingers of the right hand. Physical examination shows swelling of the palmar aspect of the wrist. Radiographs of the hand and wrist show no abnormalities. During surgical exploration, rice bodies are identified within the carpal canal as well as granuloma formation around the flexor tendons. Which of the following is the most appropriate next step?
(A) Acid-fast cultures of the tenosynovium
(B) CT scan of the abdomen
(C) Initiation of a course of allopurinol
(D) Potassium hydroxide preparation of tenosynovium
(E) Sampling of tenosynovium for amyloid

A

(A) Acid-fast cultures of the tenosynovium

Tuberculosus tenosynovitis is the most frequent tuberculosus infection in the hand.

107
Q

Most common tuberculosis infection of the hand

A

Tuberculosus tenosynovitis is the most frequent tuberculosus infection in the hand.
It may clinically simulate rheumatoid tenosynovitis. The most common pathogen is Mycobacterium marinum.

108
Q

Diagnosis of tuberculous tenosynovitis

A

Radiographs often show no abnormalities and pathology alone will often show only nonspecific tenosynovitis, but granulomas may be present. Cultures must be requested at 30 degrees Celsius to identify M. marinum.

There are several diagnostic clues for tuberculous tenosynovitis, such as proliferative synovitis occurring in the absence of a known rheumatoid or collagen vascular disease as well as the presence of rice bodies during surgical exploration. Rice bodies are infected villous bodies on the synovial surface that break off and become trapped within the inflammatory mass.

109
Q

Palmar fibrosis and arthritis: Systemic imaging

A

CT scan of the abdomen is sometimes recommended for women with palmar fibromatosis and arthritis because this condition may be associated with ovarian cancer.

110
Q

A 38-year-old man comes to the office because he has had dorsal erythema and stiffness of the metacarpophalangeal joint of the right long finger since he punched another man in the mouth during a fistfight four days ago. Radiographs of the hand show no focal findings. Which of the following is the most appropriate next step in management?
(A) Splinting and elevation
(B) Oral administration of an antibiotic
(C) Intravenous administration of an antibiotic
(D) Surgical drainage of the subcutaneous abscess
(E) Surgical drainage of the joint

A

(E) Surgical drainage of the joint

111
Q

Management of a human bite to the hand without sign of infection

A

If there are no signs of infection, it is prudent to explore the wound to rule out injury to the extensor tendon, joint capsule, and cartilage. To do so, the hand must be examined through the entire range of motion, particularly with the fingers flexed as in a clenched fist position. If there is no injury to the deeper structures, the wound can be washed out and the skin left open to heal by second intention. The patient can then be discharged with prophylactic antibiotics to cover the most common bacteria isolated from human bite wounds, i.e., Staphylococcus aureus, followed by Streptococcus species, Corynebacterium species, and Eikenella corrodens. Follow-up within 24 to 48 hours isimportant in outpatient management of these injuries. If injury is noted to the tendon, joint capsule, or joint cartilage, then operative washout and admission for intravenous antibiotics is appropriate.

112
Q

Management of a human bite to the hand with signs of infection

A

When the patient presents with a well-developed infection, antibiotic therapy alone is insufficient. Careful exploration must be performed to rule out injury and infection to deeper structures, most importantly the joint. Unrecognized or untreated pyarthrosis can lead to joint destruction and osteomyelitis. Inadequate exploration in the emergency department is usually due to inadequate anesthesia. Formal washout and drainage is sometimes best performed in the operating room.

113
Q

Management of edema in hand infections (outside of other considerations)

A

Elevation and immobilization are important in the management of edema in patients with hand infections.

114
Q
In a patient with infection of the index finger, drainage is most likely to flow proximally into which of the following spaces?
(A) Midpalmar space
(B) Parona’s space
(C) Radial bursa
(D) Thenar space
(E) Ulnar bursa
A

(D) Thenar space

115
Q

Infections of the index finger drain into:

A

Infections of the index finger drain from the flexor sheath proximally into the thenar space

116
Q

The ______ finger(s) drain into the midpalpar space

A

Infections of the long, ring, and small fingers drain into the midpalmar space.

117
Q

The ______ and _______ communicate by Parona’s space

A

The radial and ulnar bursae communicate by Parona’s space, which lies deep to the pronator muscle.

118
Q
A 35-year-old woman with hidradenitis suppurativa and cellulitis affecting the axillae is scheduled to undergo incision and drainage and initiation of antibiotic therapy. In addition to Staphylococcus aureus, which of the following organisms is most commonly isolated in patients with hidradenitis? 
(A) Peptostreptococcus sp.
(B) Pseudomonas aeruginosa
(C) Staphylococcus epidermidis
(D) Streptococcus faecalis
(E) Viridans streptococcus
A

(E) Viridans streptococcus

119
Q

Most commonly isolated organisms in patients with hidradenitis

A

Staphylococcus aureus and viridans streptococcus are the two most commonly isolated organisms in hidradenitis suppurativa.

120
Q
A 6-year-old boy with sudden onset of fever and septic shock has disseminated intravascular coagulation. Physical examination shows hemorrhagic necrosis of the skin of the upper and lower extremities. Which of the following is the most likely causative organism?
(A) Escherichia coli
(B) Group A beta-hemolytic streptococcus
(C) Neisseria meningitidis
(D) Staphylococcus aureus
(E) Varicella-zoster viru
A

(C) Neisseria meningitidis

121
Q

Organisms implicated in purpura fulminans

A

Purpura fulminans is associated with endotoxin-producing bacteria. Although Neisseria meningitidis is the most common causative organism, Streptococcus pneumoniae, Haemophilus influenzae, and Rickettsia have also been implicated.

122
Q

Presentation of purport fulminans

A

Purport fulminans manifests as severe hemorrhage and necrosis of skin associated with disseminated intravascular coagulation. Affected patients have petechial rashes, which progress to confluent areas of ecchymosis, and then to necrotic, hard, full-thickness eschar. Bilateral symmetric gangrene of the extremities necessitates amputation in as many as 20% of patients. Reconstructive procedures, including free tissue transfer, are frequently required to resurface necrotic areas and salvage extremities with exposed joints. Septic shock and organ failure may also result. Mortality rates have been reported to be as high as 60%.

123
Q

Proposed mechanism of purpura fulminans

A

The mechanism of this condition is believed to be liposaccharide-mediated endothelial damage caused by bacteria, leading to decreased serum levels of proteins C and S.

124
Q

A construction worker has an abscess of the palm of the nondominant hand after sustaining a puncture wound to the palm. In this patient,the midpalmar space is defined by which of the following boundaries?
(A) Flexor tendons, abductor pollicis muscle, superficial aponeurosis, and septum from the second metacarpal bone to the flexor digitorum profundus sheath
(B) Flexor tendons, metacarpal bone and interosseous fascia, septum from the third metacarpal to the flexor digitorum profundus sheath, and hypothenar eminence
(C) Flexor tendons, superficial palmar aponeurosis, and thenar and hypothenar eminences
(D) Flexor tendons, thenar eminence, septum from the second metacarpal bone to the flexor digitorum profundus tendon, and superficial aponeurosis
(E) Septum from the first metacarpal bone to the superficial aponeurosis, septum from the third metacarpal to the flexor tendon sheath, and lateral and medial edges of the abductor pollicis muscle

A

(B) Flexor tendons, metacarpal bone and interosseous fascia, septum from the third metacarpal to the flexor digitorum profundus sheath, and hypothenar eminence

125
Q

Midpalmar space

A

The midpalmar space is located deep to the flexor tendon. It extends dorsally to the fascia over the second and third volar interossei and the third and fourth metacarpals.The midpalmar space is bordered radially by a fascial septum extending from the third metacarpal to the flexor sheath of the flexor digitorum profundus tendon of the long finger, and ulnarly by the fascia of the hypothenar musculature. The proximal margin of the midpalmar space is a thin layer of fascia that lies just distal to the carpal canal. Thedistal margin of the midpalmar space is bordered by vertical septa of the palmar fascia, which extend almost to the web space

126
Q

Presentation of patients with infection of midpalmar space

A

In patients with infection of the midpalmar space, diagnosis is often delayed. Affected patients typically exhibit swelling of the dorsal aspect of the hand, loss of palmar concavity, and difficulty extending and flexing the fingers. Marked tenderness in the midpalmar area is characteristic, and cellulitis is often associated.

127
Q

Thenar space

A

The thenar space is located radial to the vertical septum between the third metacarpal and the flexor digitorum profundus tendon of the long finger; it extends to the radial edge of the abductor pollicis brevis tendon.

128
Q

Hyporhenar space

A

The hypothenar space contains the hypothenar muscles and is enveloped within the fascia of these muscles. It is bordered radially by a fascial septum extending from the fifth metacarpal bone to the palmar fascia.

129
Q
A 47-year-old woman has a low-grade fever, chills, and pain and swelling of the proximal interphalangeal joint of the index finger. On examination, active and passive motion of the joint produces pain. There is no lymphangitis or lymphadenopathy. Which of the following is the most likely causative organism?
(A) Eikenella corrodens
(B) Neisseria gonorrhoeae
(C) Serratia marcescens
(D) Staphylococcus aureus
(E) Viridans streptococcus
A

(D) Staphylococcus aureus

130
Q

First and second most common causative organisms of septic arthritis

A

Serratia species are a frequent cause of infection in persons who abuse intravenous drugs as well as patients with diabetes mellitus or immune system compromise.

131
Q

Serratia in hand infections

A

Staphylococcus aureus is an anaerobic gram-positive coccus that ispresent on the skin and is a frequent cause of skin and soft-tissue infections. It is the most common cause of septic arthritis of the hand and wrist.

Streptococcus species, including viridans streptococcus, are the second most common causative organism in patients with septic arthritis of the hand.

132
Q

A 50-year-old man has had the fingernail deformity shown in the photograph above for the past year. There is no history of trauma to the finger. Which of the following is the most appropriate management?
(A) Topical administration of neomycin ointment twice daily
(B) Oral administration of ciprofloxacin 400 mg twice daily
(C) Oral administration of terbinafine 250 mg daily
(D) Resection of the involved sterile matrix and grafting from the matrix of the great toe
(E) Surgical removal of the nail plate and stenting of the eponychial fold with nonadherent gauze

A

(C) Oral administration of terbinafine 250 mg daily

Topical or oral administration of antibiotics would not be expected to improve this fungal infection. In addition, topical antibiotics may aggravate the nail matrix. Resection of the sterile matrix and replacement with a graft is associated with a high incidence of recurrence and morbidity. Removal of the nail would not eliminate the fungal infection within the underlying matrix

133
Q

Onychomycosis

A

Onychomycosis is a dystrophic nail resulting from a fungal infection (onychomycosis). The diagnosis of onychomycosis can be confirmed by positive findings on fungal culture.

134
Q

Most common causes of dystrophic nails

A

Fungal infections are the underlying cause of dystrophic nails in approximately 50% of affected patients; the remaining 50% are caused by other factors, including psoriasis, lichen planus, and trauma.

135
Q

Treatment of onychomycosis

A

Terbinafine and itraconazole have offered new treatment options. This agents are administered for six weeks, and hepatic function is monitored only in those patients who have a history of hepatitis, liver disease, or heavy alcohol use.

136
Q

Adverse effects of Terbinafine

A

adverse effects associated with terbinafineuse include Stevens-Johnson syndrome, neutropenia, hepatotoxicity, hepatic failure, erythema multiforme, toxic epidermal neurolysis, and anaphylaxis. In addition, terbinafine is far more costly than previously used antifungal agents.

137
Q

A 34-year-old man has had pain and swelling of the long and ring fingers for the past three days. On physical examination, there is a sausage-like appearance of the fingers. The patient has pain on passive stretch of the fingers, and there is tenderness over the flexor tendon sheaths. Radiographs show swelling of the soft tissues. In addition to intravenous administration of antibiotics, which of the following is the most appropriate management?
(A) Needle aspiration of the flexor tendon sheaths
(B) Incision into the fingers
(C) Incision into the joints
(D) Incision into the palm
(E) Opening and irrigation of the flexor digital sheath

A

(E) Opening and irrigation of the flexor digital sheath

138
Q

Kanavel’s signs

A

Kanavel’s four signs (fusiform swelling, partial flexed posturing of the finger, tenderness over the flexor tendon sheath, and pain with passive extension of the finger).

139
Q

Incisions/approach for flexor tenosynovitis

A

he surgeon should make an incision into the palm that is sufficiently wide to allow for access to and visualization of the proximal aspect of the A1 pulley. Another incision is made distally to allow access to the A4 and A5 pulleys. If necessary, the incisions can be extended distally and/or proximally to treat infected, necrotic tissue.

In patients with advanced flexor tenosynovitis, the flexor digital sheath is typically distended with purulent material. A plastic irrigation catheter is inserted at the level of the A1 pulley and threaded distally into the sheath to allowfor irrigation beyond the A5 pulley. This technique will facilitate complete decompression of the sheath without sacrificing the pulleys. Irrigation can be provided via the catheter continuously for as long as 24 hours, depending on the severity of infection.

140
Q

Early flexor tenosynovitis

A

Needle aspiration is useful in establishing a diagnosis of or treating early tenosynovitis (characterized by the presence of only one or two of Kanavel’s signs) or as initial treatment during pregnancy.

141
Q
An otherwise healthy 48-year-old nurse is brought to the emergency department because she has intense pain in the right lower extremity after sustaining a minor abrasion of the right knee. She has undergone evaluation twice within the past 48 hours for pain disproportionate to the level of injury. Temperature is 38.8 C (102 F) and blood pressure is 70/50 mmHg. On examination, the extremity is warm, swollen, and erythematous. There is a bluish blister at the site of injury. Laboratory studies show an increased leukocyte count, decreased platelet count, increased serum creatinine level, and increased international normalized ratio (INR). Radiographs show no abnormalities.Which of the following is the most likely diagnosis?
(A) Clostridium necrotizing fasciitis
(B) Cutaneous anthrax
(C) Pseudomonas ecthyma gangrenosum
(D) Staphylococcal cellulitis
(E) Streptococcal toxic shock syndrome
A

(E) Streptococcal toxic shock syndrome

142
Q
A 40-year-old man has a painful, fluctuant abscess over the dorsal aspect of the left hand at the level of the metacarpophalangeal joints. On physical examination, the index finger is abducted away from the long finger. This abscess most likely courses through which of the following anatomic sites?
(A) Extensor tendon sheath
(B) Flexor tendon sheath
(C) Palmar bursa
(D) Palmar fascia
(E) Parona’s space
A

(D) Palmar fascia

This patient has a collar button abscess, which communicates from the volar web space to the dorsal aspect of the hand via the palmar fascia or lumbrical canal.

143
Q

Collar button abscess

A

This patient has a collar button abscess, which communicates from the volar web space to the dorsal aspect of the hand via the palmar fascia or lumbrical canal. Finger abduction isa characteristic finding. Appropriate management is drainage of the abscess using a combined volar and dorsal approach.

144
Q

A 25-year-old man is brought to the emergency department four hours after being bitten by araccoon. He has not been previously vaccinated for rabies. In addition to irrigation of the wound, which of the following is the most appropriate management?

A) Rabies immunoglobulin into the deltoid; vaccination into the gluteus
B) Rabies immunoglobulin into the gluteus; vaccination into the deltoid
C) Rabies immunoglobulin into the subcutaneous tissue of the thigh; vaccination into the gluteus
D) Rabies immunoglobulin into the wound margin; vaccination into the deltoid
E) Rabies immunoglobulin into the wound margin; vaccination into the wound margin

A

D) Rabies immunoglobulin into the wound margin; vaccination into the deltoid

145
Q

Vaccination for a patient who has been bitten by a potentially rabid animal and who has not been previously vaccinated

A

In a patient who has been bitten by a potentially rabid animal and who has not been previously vaccinated, management should include wound care and administration of both rabies immune globulin (RIG) and rabies vaccine. Because rabies incubation periods of more than one year have been reported in humans, the prophylactic regimen should be initiated immediately in any person who has been bitten by an animal with suspected or proven rabies regardless of the length of the delay, as long as clinical signs of rabies are not present. Studies have shown that a regimen of one dose of RIG and five doses of human diploid cell vaccine (HDCV) over a 28-day period is a safe treatment protocol that induces an excellent antibody response

146
Q

Rabies postexposure prophylaxis schedule, not previously vaccinated

A

Wond cleansing - All postexposure treatment should begin with immediate thorough cleansing of allwounds with soap and water. If available, a virucidal agent such povidone-iodine solution should be used to irrigate the wounds

RIG - Administer 20 IU/kg body weight. If anatomically feasible, the full dose should be infiltrated around the wound(s) and any remaining volume should be administered IM at an anatomical site distant from vaccine administration. Also, RIG should not be administered in the same syringe as vaccine. Because RIG may partially suppress active production of antibody, no more than the recommended dose should be given

Vaccine - HDCV, RVA, or PCEC 1.0 mL, IM (deltoid area +), one each on days 0¤, 3, 7, 14 and 28.

147
Q

Why should no more than the recommended dose of RIG should be given

A

Because RIG may partially suppress active production of antibody, no more than the recommended dose should be given

148
Q

Rabies postexposure prophylaxis schedule, previously vaccinated

A

Wound cleansing - All postexposure treatment should begin with immediate thorough cleansing of all wounds with soap and water. If available, a virucidal agent such povidone-iodine solution should be used to irrigate the wounds.

RIG - should not be administered

Vaccine - Vaccine HDCV, RVA, or PCEC 1.0 mL, IM (deltoid area +), one each on days 0¤ and 3

149
Q

Site of administration for rabies vaccine

A

The deltoid area is the only acceptable site of vaccination for adults and older children. For younger children, the outer aspect of the thigh may be used. Vaccine should never be administered in the gluteal area.

150
Q

A 21-year-old woman has swelling and edema of the left index fingertwo days after sustaining a puncture wound to the finger. Which of the following is the most sensitive indicator of bacterial flexor tenosynovitis in this patient?
(A) Diffuse erythema of the finger
(B) Drainage from the wound
(C) Fusiform swelling of the finger
(D) Pain on passive extension of the finger
(E) Tenderness along the flexor tendon sheath

A

(D) Pain on passive extension of the finger

151
Q

Most sensitive test for flexor tenosynovitis

A

the fourth classic Kanavel sign, pain with passive extension of the digit, is the most sensitive test for flexor tenosynovitis.

152
Q
A 49-year-old man with type 2 diabetes mellitus has had a "sausage" appearance of the left long finger from the metacarpophalangeal joint to the fingertip for the past two days. The finger is held in flexion at rest. On physical examination, there is tenderness along the volar aspect of the finger, and the patient has pain with passive extension.Which of the following is the most likely diagnosis?
(A) Cellulitis
(B) Felon
(C) Osteomyelitis
(D) Paronychia
(E) Tenosynovitis
A

(E) Tenosynovitis

153
Q

Most common causative organism of tenosynovitis

A

Staphylococcus aureus is the most likely causative organism.

154
Q

Cellulitis of the hand

A

Cellulitis is a common superficial infection that typically affects the dorsal aspect of the hand and is characterized by erythema, edema, and lymphangitis. Beta-hemolytic streptococcus is most frequently associated

155
Q

Felon

A

Felons are infections of the pulp space (which is compartmentalized by septa) typically caused byStaphylococcus aureus. Although tenosynovitis may develop in a patient with an advanced felon, the infection is more likely to be localized at the pulp initially, and the patient would have throbbing pain, especially when the finger is placed in a dependent position.

156
Q

Osteomyelitis of the hand

A

Patients with osteomyelitis have localized pain, swelling, and erythema along the course of one of the long bones of the hand. This condition often develops secondary to localized infection by hematogenous spread.

157
Q

A 55-year-old woman has had pain, swelling, and erythema of the left arm for the past 24 hours. She underwent mastectomy and axillary lymph node dissection on the left four years ago. On examination, she is afebrile. Laboratory studies show a leukocyte count that is within normal limits. Which of the following is the most appropriate management?
(A) Lymphatic massage
(B) Application of a compression bandage and elevation of the extremity
(C) Topical application of an antibiotic
(D) Intravenous administration of an antibiotic
(E) Incision and drainage

A

(D) Intravenous administration of an antibiotic

In this patient who has had the spontaneous onset of cellulitis of the arm after undergoing axillary lymph node dissection, the most appropriate management is intravenous administration of an antistreptococcal antibiotic. Fever and leukocytosis are typically associated with cellulitis but are not required to make the diagnosis, as many of these patients will be afebrile and will not have an increased leukocyte count or absolute neutrophil count on serologic testing. Anti-streptolysin O titer may be positive

158
Q
Anotherwise healthy 44-year-old woman has chronic, persistent paronychia of the index finger. Administration of oral and topical antifungal agents has not resulted in improvement of symptoms. Which of the following is the most appropriate management?
(A) Incision and drainage
(B) Removal of the nail plate
(C) Eponychial marsupialization
(D) Obliteration of the nail matrix
(E) Amputation of the fingertip
A

(C) Eponychial marsupialization

159
Q

Chronic paronychia

A

Chronic paronychia is a recurrent abscess beneath the eponychial edge of the fingernail associated with repeated exposure to a moist environment. In patients with this condition, Candida albicans is the most frequently cultured organism. Chronic paronychia can evolve following an acute episode if drainage of the abscess is inadequate or inappropriate antimicrobial agents are prescribed.

160
Q

Treatment of chronic paronychia

A

If the infection does not respond to treatment, radiographs should be obtained to determine bony involvement. In addition, biopsy specimens and cultures of soft tissue and/or bone may be indicated to identify the pathology of the condition, as the underlying cause may be a misdiagnosed malignant tumor. If radiographs, biopsy specimens, and cultures show no disease, appropriate management is excision of the thickened dorsal nail roof (ie, marsupialization), typically a crescent-shaped piece with a width of 3 to 5 mm

161
Q

Treatment of acute paronychia w/drainage beneath the plate

A

Removal of a segment of nail plate is indicated in patients with acute paronychia if there is drainage beneath the plate.

162
Q

Treatment of paronychia that appears secondary to fungal infection

A

In a patient who has paronychia that appears to be caused by a fungal organism, management should focus on administering oral and topical antifungal agents, such as itraconazole and ketoconazole, and minimizing the moist environmental conditions that have predisposed the patient to the infection

163
Q

A 34-year-old woman undergoes laser-assisted liposuction of the abdomen, hips, and inner and outer thighs using a tumescent technique. A total of 2500 mL of aspirate is removed. Postoperative recovery is uneventful, and the patient is discharged home the same day. She comes to the emergency department 4 days later with intense pain over the lower abdomen and flanks. Temperature is 101°F (38.3°C). Physical examination shows the skin has well-demarcated erythema, induration, and bullae forming at multiple sites. Her incisions are seeping clear, grey fluid. Which of the following is the most appropriate management?
A) Exploratory laparotomy for presumed bowel perforation
B) Intravenous administration of antibiotics and local burn care
C) Intravenous administration of antibiotics only
D) Oral administration of antibiotics
E) Surgical debridement of the involved tissue

A

E) Surgical debridement of the involved tissue

Necrotizing fasciitis is a rapidly progressive soft-tissue infection characterized by necrosis of the fascia and subcutaneous fat with subsequent necrosis of the overlying skin. Although complication rates associated with liposuction are not unduly increased, infection is a major concern, and cases of prolonged inflammation, septic shock, and infections have been documented. Likewise, cases of necrotizing fasciitis following liposuction have been reported on several occasions and, according to data reported in the literature, the overall incidence of necrotizing fasciitis is equal to 0.4 per 100,000 patients.

There are two common forms that are reported: infections caused by Streptococcus pyogenes and mixed infections caused by a variety of microbes, including Escherichia coli, Proteus, Serratia, and Staphylococcus aureus. A detailed case of necrotizing fasciitis sustained by Mycobacterium chelonae after a combined procedure of liposuction and lipofilling has also been described. The progressive necrosis of the tissues typically involves the superficial fascia and the subcutaneous layer, but is limited in extension to the skin; the extent of the gangrene at the fascial layer is usually more severe and greater than at the skin level.

Necrotizing fasciitis is virtually unnoticeable in the first 48 hours with nonspecific symptoms. In the days that follow, an extensive, hardened region forms, which is often dark in the center. Severe pain and necrosis follow at the level of the infection.

Drainage of “dishwater fluid” is often pathognomonic. Metabolic changes occur, ending with respiratory distress, oliguria, acidosis, increased troponin concentrations, and sepsis. Diagnosis and treatment consist of surgical exploration and debridement that reveal necrotic, edematous, subcutaneous fat. Bacteriologic analysis of exudate, cultures, and histologic evaluation complete the diagnosis.

Early diagnosis is imperative to avoid a fatal outcome. Treatment is based on immediate and aggressive surgical debridement with combined antibiotic therapy. Because necrotizing fasciitis is a progressive, rapid infection, the wound typically is left open for a planned “second-look” operation and additional debridement if necessary. The mortality rates are increased and range up to 70% but decrease to 4.2% after immediate surgical intervention. Delay in debridement increases mortality.

164
Q

A 40-year-old woman with chronic hidradenitis suppurativa of the bilateral axillae is referred for surgical management after 4 years of local wound care, multiple corticosteroid injections, and oral antibiotics failed to improve her symptoms. Physical examination shows multiple inflamed nodules and draining sinuses. Photographs are shown. Which of the following treatment options is most likely to provide long-term resolution of this patient’s symptoms?

A) Axillary lymphadenectomy and coverage with a trapezius flap
B) Carbon dioxide laser and silver sulfadiazine
C) Fistulotomy and negative pressure wound therapy
D) Lesion excision and primary closure
E) Total axillary excision and skin grafting

A

E) Total axillary excision and skin grafting

Hidradenitis is a chronic and often disabling skin condition of the apocrine glands. It results in abscesses, inflammatory nodules, and draining sinuses in the axillae, groin, perineum, and occasionally the submammary area. It affects approximately 1 in 300 people, and it is more common in women, people who smoke, and people who are overweight. The axillae are more commonly involved in women, and the perineum more commonly in men.

Early treatment centers around local wound care, oral antibiotics, corticosteroid injections, hygiene, weight loss, and smoking cessation. Decreased estrogen levels may also contribute, and disease often subsides with oral contraceptives and during pregnancy. Unresponsive or extensive disease requires surgical intervention for long-term resolution. Relapse rates are high, and healing is often slow. Although limited excision and closure of solitary lesions is an effective short-term treatment, new lesions are likely to develop. Fistulotomy alone has an unacceptably high recurrence rate.

A long-term resolution in this severely affected patient requires complete excision of the hair-bearing skin of the affected area. The large defect created needs broad skin coverage. The best options include split-thickness skin grafting or grafting with negative pressure wound therapy immobilization. Negative pressure wound therapy followed by healing by secondary intention or coverage with local flaps are additional options, though scar contractures and shoulder stiffness are risks.

Laser treatments are too superficial to treat the deeper apocrine structures. Lymphadenectomy is not necessary for a cure.

165
Q
A 48-year-old right-hand–dominant man with a history of cadaveric renal transplantation comes to the emergency department because of a swollen, painful left hand. A photograph is shown. He works as a crab fisherman and reports that he was bitten in the first web space by a crab 18 hours ago. He is taken to the operating room for emergent debridement. Intraoperative Gram stain shows a gram-negative, curve-shaped rod. In addition to aggressive surgical treatment, administration of which of the following antibiotics is most appropriate to treat this patient?
A) Cephalexin
B) Levofloxacin
C) Linezolid
D) Nafcillin
E) Vancomycin
A

B) Levofloxacin

Vibrio vulnificus is a common Vibrio species causing soft-tissue infections of the hand. Vibrio species are ubiquitous in aquatic environments including saltwater bodies. Immunocompromised hosts are at greater risk for amputation and death. The best chance for patient survival includes early diagnosis and initiation of appropriate antibiotics, as well as urgent surgical debridement for any evidence of necrotizing infection.

A broad range of antibiotics are effective against Vibrio species, including V. vulnificus. Agents effective against gram-negative rods, including quinolones, aminoglycosides, and aminopenicillins, are all effective against V. vulnificus. While all cephalosporins are effective against Vibrio species, third-generation cephalosporins are up to 130 times more potent than first- or second-generation cephalosporins. Vancomycin, nafcillin, and linezolid are all narrow-spectrum antibiotics, primarily effective against gram-positive cocci, and are not effective against Vibrio species.

166
Q

A 55-year-old man with a history of smoking and poorly controlled diabetes mellitus comes to the emergency department because of a 12-hour history of unilateral proptosis, impaired vision, and severe headaches. Intranasal examination shows dried crusting and black discoloration of the lateral nasal wall and turbinates. Radiology shows a 4-cm mass within the maxillary sinus that extends into the orbit. Biopsy of the mass shows nonseptate hyphae. Which of the following is the most appropriate next step in management?
A) Craniofacial resection and free flap
B) Emergent debridement of the sinuses and orbital exenteration
C) Intravenous administration of an antibiotic
D) Outpatient oral antifungal medications
E) Referral to medical oncology

A

B) Emergent debridement of the sinuses and orbital exenteration

Rhinocerebral mucormycosis is a rare opportunistic infection of the sinuses, nasal passages, oral cavity, and brain caused by saprophytic fungi. The infection can rapidly result in death. Rhinocerebral mucormycosis commonly affects individuals with diabetes and those in immunocompromised states. The diagnosis of mucormycosis is established by obtaining a biopsy specimen of the involved tissue, and frozen tissue samples should be immediately evaluated for signs of infection. Microscopic characterization of non-septate hyphae, rhizoids, columellae, sporangia, and sporangiospores helps to define genus and species within the order Mucorales. Optimal therapy requires a multidisciplinary approach that relies on prompt institution of appropriate antifungal therapy with amphotericin B (AmB), reversal of underlying predisposing conditions, and, where possible, surgical debridement of devitalized tissue. Surgery should be considered early, and if possible, emergently with the goal of removing all necrotic tissue. Repeated debridements are frequently necessary and the extent of surgery should ideally be guided by evaluation of frozen tissue sections examined histologically. In the scenario provided, orbital involvement will most likely require sacrifice of the eye.

167
Q
A 15-year-old girl comes to the office because of a 1-day history of infection of the right index finger. Physical examination shows the tip of the finger is tender and swollen over the pulp. There is no history of trauma. Which of the following organisms is the most likely cause of this patient’s condition?
A) Candida albicans
B) Eikenella corrodens
C) Listeria monocytogenes
D) Pasteurella multocida
E) Staphyloccus aureus
A

E) Staphyloccus aureus

Staphylococcus is still the most common organism in hand infections. The most common in felons is Staphylococcus aureus. Methicillin-resistant Staphylococcus aureus community-acquired (MRSA-CA) infections are now the most predominant strain in hand infections, comprising 60% of Staphylococcus aureus infections.

Pasteurella multocida should be considered with most animal bites, although it is most common with cat bites. Eikenella corrodens is associated with human bites. There is no history of bites in this case.

Listeria monocytogenes has been reported in flexor tenosynovitis in immunocompromised patients.

Candida albicans is usually associated with chronic paronychia.

168
Q
A 55-year-old man undergoes microsurgical replantation of an amputated ear. There is venous congestion, and leeches are applied. This patient is at risk for infection by which of the following organisms?
A) Actinobacillus lignieresii
B) Aeromonas hydrophila
C) Eikenella corrodens
D) Pasteurella canis
A

B) Aeromonas hydrophila

Aeromonas hydrophila is an organism present in the leech species Hirudo medicinalis gastrointestinal tract that can lead to an infection if used medicinally. In this patient with venous congestion and application of leeches, antibiotic prophylaxis is recommended with fluoroquinolones, tetracycline, or trimethoprim-sulfamethoxazole.

Actinobacillus lignieresii is seen in horse bites, Pasteurella canis in dog bites, and Eikenella corrodens in human bites.

In a recent review of ear reattachment methods, a variety of approaches have been used including microsurgical reattachment, burying of the part in a subcutaneous pocket, periauricular tissue flaps for coverage of the part, and direct reattachment as a composite graft. Microsurgical replantation is associated with the best aesthetic outcome even if venous anastomosis is not possible and leeching is necessary.

169
Q

A 35-year-old woman with type 2 diabetes mellitus is evaluated in the emergency department because of severe pain and drainage from the right buttock 36 hours after undergoing bilateral buttock augmentation with autologous fat harvested from the thighs. Temperature is 102.0°F (38.9°C), heart rate is 105 bpm, respiratory rate is 16/min, and blood pressure is 90/60 mmHg. Physical examination of the right buttock shows brawny erythema and drainage of turbid fluid from an injection site. The patient has marked tenderness of the buttock, and the abdomen is nontender. White blood cell count is 18.5 × 109/L and serum creatinine concentration is 1.5 mg/dL. After resuscitation, which of the following is the most appropriate next step in management?
A) CT scan of the abdomen and pelvis
B) Inpatient intravenous antibiotic therapy
C) Outpatient oral antibiotic therapy
D) Surgical exploration of the wound
E) Ultrasonography of the buttock

A

D) Surgical exploration of the wound

The most appropriate next step in management is to surgically explore the wound in the operating room. The clinical picture is of a severe, rapidly progressing infection, possibly necrotizing fasciitis. A high index of suspicion and early treatment are vital for successful outcomes. Necrotizing fasciitis is a rare and rapidly progressive infection of the deeper layers of skin and subcutaneous tissues, easily spreading across the superficial fascial plane, with subsequent death of the overlying skin and severe systemic toxicity. Liposuction is the most frequently associated cosmetic surgery with this infection. Signs and symptoms are insidious, nonspecific, or virtually unnoticeable early in the course of the disease. Later, erythema, prominent edema, and induration appear, accompanied by intense or intolerable pain. The clinical picture evolves into systemic toxicity and eventually multiple organ failure. Risk factors for necrotizing fasciitis include diabetes mellitus, immunosuppression, age older than 50 years, malnutrition, and peripheral vascular disease.

There are two forms of the disease: one caused by Streptococcus pyogenes, and the other by mixed infections caused by a variety of microbes, including Escherichia coli, Proteus, Serratia, and Staphylococcus aureus. The progressive necrosis of the tissues typically involves the superficial fascia and the subcutaneous layer. The extent of the gangrene at the fascial layer is typically more severe and greater than at the skin level. This insidious infection is virtually unnoticeable and nonspecific in the first 24 to 48 hours; however, in the following days, an extensive, hardened area appears, which is often dark colored in the center. Intense pain and skin necrosis follow at the level of the infection. Metabolic changes occur, ending with respiratory distress, oliguria, acidosis, increased creatine kinase activity, increased troponin concentrations, and toxic syndrome. Diagnosis and treatment consists of surgical exploration and debridement that reveal necrotic, edematous subcutaneous fat. Bacteriologic analysis of exudate, cultures, and histologic evaluation complete the diagnosis.

Early diagnosis is imperative to avoid a fatal outcome. Because necrotizing fasciitis is a progressive, rapid infection, a staged “second-look” operation and, if necessary, additional debridement should be performed. The mortality rates are high and range from 20 to 70%, but decrease to 4.2% after immediate surgical intervention.

Though antibiotic therapy is an integral part of the treatment, surgical exploration is key. CT scans and ultrasonography will not change the treatment plan and are therefore not the appropriate next step in management.

170
Q

An otherwise healthy 35-year-old man comes to the emergency department because of a 2-day history of swelling and pain in the index and long fingers of the right hand. He reports pain when he attempts to flex these fingers or bring them together. Physical examination shows no pain with passive extension of the fingers or during axial loading. There is pain with passive adduction of the fingers. A photograph is shown. Which of the following is the most appropriate management?
A) Arthrotomy of the metacarpophalangeal joint of the long finger
B) Division of ulnar-sided Cleland ligament of the index finger
C) Dorsal and volar incisions in the proximal second web space
D) Drainage of the mid-palmar space
E) Release of the A1 pulley of the index and long fingers

A

C) Dorsal and volar incisions in the proximal second web space

The patient described has a web space (collar-button) abscess. Pus resides dorsal to and volar to the natatory fibers of the palmar fascia with a small connection between the two spaces passing through the natatory fibers. It is drained through proximal dorsal and volar incisions in the web space. One should not incise through the apex of the web space, as this may lead to a web space contracture.

Incisions are allowed to heal by secondary intention.
Cleland ligament is a separate portion of the palmar fascia located within the finger dorsal to the neurovascular bundle. It is distal to the purulence in a web space abscess. Absence of pain with axial loading of the digits makes a joint space infection unlikely, so drainage of the metacarpophalangeal joint is unnecessary. Some approaches to drainage of flexor tenosynovitis involve release of the proximal sheath through the A1 pulley; the appearance in the photo (lack of fusiform swelling), and absence of pain with passive extension of the fingers, make flexor tenosynovitis unlikely in this patient. Infection of the mid-palmar space would produce more proximal pain and swelling in the palm, which is not present in this patient.

171
Q

A 21-year-old man with type 1 diabetes mellitus comes to the emergency department because of a large necrotizing, non-purulent infection after minimal trauma to the right cheek. Radical surgical debridement of the ulcer is performed, and the tissue is sent for histologic and microbiologic evaluation. Which of the following organisms are most likely to be found on light microscopy?
A) Chain-like collections of gram-positive bacteria
B) Grape-like clusters of gram-positive bacteria
C) Right angle nonseptate branching hyphae
D) Septate nonbranching hyphae and yeast forms
E) Tiny yeast forms with occasional unequal bud formation

A

C) Right angle nonseptate branching hyphae

Given the patient’s history of diabetes and necrotizing non-purulent infection after minimal trauma, he is likely to have mucormycosis, a life-threatening fungal infection caused by organisms from the class Zygomycetes. On microscopy, tissue samples from patients with mucormycosis demonstrate right-angle nonseptate branching hyphae.

Grape-like clusters of gram-positive bacteria is not appropriate. This option describes the characteristic appearance of a staphylococcal infection. Given the patient’s lack of cellulitis or purulent infection, it is an unlikely mechanism for this necrotizing ulceration.

Septate nonbranching hyphae and yeast forms is not appropriate. This option describes the characteristic appearance of a candidal infection. Given the patient’s lack of marked erythema and excoriation, and location of the infection on the face, rather than in skin folds, the likelihood of Candida as the primary pathogen is extremely low.

Chain-like collections of gram-positive bacteria is not appropriate. This option describes the characteristic appearance of a streptococcal infection. Although streptococcal infections are common in the head and neck region, the patient’s history and appearance of the lesion do not support Streptococcus as the causative organism.
Tiny yeast forms with occasional unequal bud formation is not appropriate. This option describes the characteristic appearance of Histoplasma capsulatum, an opportunistic fungus, which may cause marked pulmonary infections in immunocompromised patients.

Often emergent debridement is required, and that decision will need to be made on Gram stain, not on final culture.

172
Q

A 37-year-old man comes to the emergency department 4 hours after he sustained a human bite wound to the nondominant hand. Examination shows no erythema, swelling, purulent drainage, lymphangitis, or fever. Exploration of the wound shows no joint or tendon involvement. Debridement and irrigation of the wound is performed. Which of the following is the most appropriate next step in management?
A) Administration of amoxicillin-clavulanate 875/125 mg twice daily
B) Administration of clindamycin 450 mg three times daily
C) Administration of doxycycline 100 mg twice daily
D) Administration of trimethoprim-sulfamethoxazole 1 double-strength tablet twice daily
E) Observation

A

A) Administration of amoxicillin-clavulanate 875/125 mg twice daily

A prospective, randomized study has shown that antibiotic prophylaxis is superior to placebo in decreasing infections after human bites that are less than 24 hours old. The most common pathogens in human bite wounds are S aureus, E corrodens, H influenzae, and beta lactamase-producing anaerobic bacteria. Eikenella species are resistant to clindamycin. Meanwhile, doxycycline and trimethoprim-sulfamethoxazole are not effective against anaerobes. Of the options mentioned, only amoxicillin-clavulanate has good activity against all common oral pathogens.

173
Q

A 65-year-old woman undergoes treatment for breast cancer with intravenous doxorubicin through a subcutaneous port on the left side of the chest. The patient complains of severe pain around the port 5 minutes after the treatment is initiated, and the infusion is discontinued. Examination shows marked swelling (8 × 6 cm) and erythema of the skin. Which of the following is the most appropriate next step in management?
A) Flushing of the port with a saline solution
B) Intravenous administration of dexrazoxane
C) Removal of the port
D) Subcutaneous injection of dimethyl sulfoxide
E) Subcutaneous injection of saline solution

A

B) Intravenous administration of dexrazoxane

The most appropriate next step in management is to initiate intravenous dexrazoxane. Recent data support the use of this agent in extravasation of anthracyclines (e.g., doxorubicin) as an antidote. It has been shown to decrease the frequency and severity of tissue injury. The mechanism by which dexrazoxane diminishes tissue damage is unknown. However, two mechanisms of action are hypothesized: 1) reduction of oxidative stress due to complexes of metal ions and anthracyclines by chelating metal ions; 2) and blockade of topoisomerase II poisons by catalytic inhibition of topoisomerase II. Patients receive treatment with dexrazoxane 1000 mg/m2 administered intravenously on days 1 and 2, and 500 mg/m2 on day 3. The infusion is given in the opposite arm of the extravasation site and is administered as soon as possible and no longer than 6 hours after the extravasation accident.

Flushing the port is not indicated because the port may be malfunctioning and this action could extravasate the flushing agent. Adding more fluid to the area (extravasation of saline from the port, subcutaneous saline injection, etc.) will only add to the pressure on the overlying skin, injuring the skin further. Removal of the port is not indicated until it has been ascertained it is malfunctioning.

Dimethyl sulfoxide (DMSO) is a known treatment option for anthracycline extravasations. However, it is used as a topical agent with or without cortisone, not as a subcutaneous injection. Additionally, it should not be used in patients who are receiving dexrazoxane because it has been shown to decrease the efficacy of the dexrazoxane therapy.

174
Q
A 55-year-old man who is a commercial fisherman comes to the office because of a 3-week history of a swollen index finger. The patient holds the finger in a flexed position. Extension of the digit is difficult but not painful. Surgical exploration shows nonpurulent fluid in the tendon sheath. Culture of the fluid is most likely to show which of the following pathogens?
A) Eikenella corrodens
B) Mycobacterium marinum
C) Sporothrix schenckii
D) Staphylococcus aureus
E) Vibrio vulnificus
A

B) Mycobacterium marinum

The patient described has an occupation that exposes him to contaminated water and raw seafood. Infections from Mycobacterium marinum and Vibrio vulnificus are both possible.

Mycobacterium marinum is the most common atypical mycobacterium seen in hand infections, often seen after penetration by aquatic equipment, colonized marine life, or contact with contaminated water. The most common deep infection is flexor tenosynovitis, and may present as a chronic tenosynovitis.

Sporothrix schenckii is a fungus found in plants and soil. Inoculation with the pathogen results in a papule at the entry site, followed by development of lesions along the lymphatic chain.

Suppurative flexor tenosynovitis typically presents with pain over the tendon sheath, semiflexed position of the involved digit, pain on passive extension, and symmetrical swelling of the finger. This classic presentation is commonly caused by pathogens such as Staphylococcus aureus or Eikenella corrodens (often seen in association with human bite injuries). In tenosynovitis infections involving atypical mycobacterium, however, there is absence of pain.

Vibrio vulnificus is a species of gram-negative, motile, curved bacterium found in the coastal waters of the United States. Infections from V vulnificus may be caused by direct exposure of an open wound to warm seawater containing the organism or from handling raw seafood or marine wildlife. Infections caused by V vulnificus result in painful cellulitis that progresses rapidly and presents with marked local tissue swelling with hemorrhagic bullae. Bacteremia with systemic symptoms is commonly seen.

175
Q
A 22-year-old man who has type 1 diabetes mellitus comes to the office because of a 1-day history of cervical and chest pain. Temperature is 102.2°F (39.0°C). Physical examination shows right-sided cervical erythema and moderate swelling. A broad-spectrum antibiotic is administered, and he undergoes incision and drainage. During the procedure, extensive soft-tissue necrosis not confined by fascial planes is noted. Which of the following is the most likely cause of this infection?
A) Mastoiditis
B) Meningitis
C) Parotitis
D) Pharyngitis
E) Sinusitis
A

D) Pharyngitis

The patient described has cervical necrotizing fasciitis (CNF) that likely extends into the mediastinum. Pharyngeal and tonsillar infections, along with dental abscesses, are the most common sources of infection. Diabetes and other immunocompromised states are frequent comorbidities in this disease. Early presentation may be clinically indistinguishable from a superficial soft-tissue infection and, therefore, requires a high index of suspicion. Skin necrosis is often a late finding, as the infection begins in the fascial and deep tissues of the neck. CT scan is usually obtained to define the extent of the disease and may or may not show gas within the soft tissues. Patients are treated with broad-spectrum antibiotics and aggressive surgical debridement.

Mastoiditis is an infection of the mastoid air cells usually arising in the setting of untreated otitis media. Mastoiditis can spread to surrounding structures, including the brain.

Meningitis is an infection of the meninges that is associated with neck pain as well as fever, headache, and photophobia. It does not present with unilateral neck erythema and swelling.

Parotitis is an infection of the parotid gland that usually arises in the setting of an obstructed parotid duct and is a very rare but potential cause of CNF.

Sinusitis is not usually associated with CNF but may spread to the orbit, resulting in orbital cellulitis or orbital abscess, and, occasionally, cavernous sinus thrombosis. Cavernous sinus thrombosis is a life-threatening condition.

176
Q
A 50-year-old man comes to the office because of a 2-day history of worsening erythema, swelling, and tenderness of the interphalangeal (IP) joint of the thumb. He reports similar episodes in the past involving other joints but does not recall any history of trauma or skin breakage in the area. X-ray study of the thumb shows soft-tissue swelling only. No bony abnormalities are noted. Which of the following is the most appropriate first step in management?
A ) Administration of antibiotics
B ) Aspiration of joint fluid
C ) Elevation and splinting of the joint
D ) MRI of the joint
E ) Surgical washout
A

B ) Aspiration of joint fluid

The most appropriate first step in management in the scenario described is to aspirate fluid from the joint for analysis. There are several conditions that mimic acute hand infections, and it is important to recognize them so that the appropriate treatment can be initiated. Crystalline deposition diseases, such as gout and pseudogout, tend to present with joint swelling, erythema, pain with motion, and fever, much like the signs of infection. The only way to confirm the diagnosis is to aspirate the joint and look for crystals with polarizing microscopy. Though the description of the thumb in the scenario described is consistent with possible infection, the fact that there has been no trauma or break in the skin around the joint, and the patient’s history of acute arthritic episodes in other joints, raises some suspicions that this could be noninfectious. Therefore, joint aspiration to rule out other causes of acute arthritis is warranted. Administration of antibiotics and elevation and splinting should begin after the aspiration, if indicated. MRI of the joint and surgical washout are not indicated at this time.

177
Q

An otherwise healthy 25-year-old woman comes to the emergency department because of pain, redness, swelling, and stiffness in the long, ring, and little fingers of her right hand 36 hours after being bitten by her cat. The cat is up-to-date with rabies vaccinations. The patient does not take any medications. The patient states that she was told never to take penicillin because she “stopped breathing” when she took it as a child. Examination shows multiple small, draining puncture wounds along the proximal volar and ulnar aspects of the proximal phalanges, a collar-button abscess between the ring and little fingers, and edema over the dorsum of the hand. Pending results of culture, which of the following is the most appropriate empiric antibiotic therapy?
A ) Ampicillin-sulbactam
B ) Cefazolin
C ) Ciprofloxacin and clindamycin
D ) Tetracycline and trimethoprim-sulfamethoxazole
E ) Trimethoprim-sulfamethoxazole

A

C ) Ciprofloxacin and clindamycin

For the patient described with a penicillin or cephalosporin allergy, trimethoprim-sulfamethoxazole (TMP/SMX) and clindamycin or metronidazole, or a fluoroquinolone and clindamycin or metronidazole, are good antibiotic therapy combinations.

Cat bites account for 5 to 15% of animal bites and occur more often in women and adults (median age is 20 years). Most cat bite injuries occur on the upper extremity with ‘scratches’ on the finger or hand. Infection rates after cat bites range from 50 to 75%. This is typically attributed to the fine, sharp teeth of cats, which produce puncture wounds and penetrate deeply into the soft tissues and joint capsules of the hand.

Wounds to the hand demonstrate an increased risk of infection; closed fist injuries have the highest risk. Initial medical management of bite wounds includes ascertaining the rabies status of the animal and updating the tetanus immunization of the patient if necessary. If rabies is suspected, irrigation with povidone-iodine solution will reduce the development by up to 90%. A careful examination of the affected hand/extremity should address the surrounding soft tissues and potential spaces of the hand, tendons, joint and joint capsules, and underlying bone. The wound should be irrigated with saline after obtaining wound cultures. Necrotic tissue should be debrided, and any cat bite with worsening pain will require incision and irrigation of the puncture site. All cat bite wounds should be left open, although large gaping wounds can be loosely approximated. Before definitive surgical treatment of any associated abscess or debridement of devitalized tissue, empiric antibiotic therapy should be started.

The majority of cat bite wounds are polymicrobial and the anaerobic organisms are often overlooked. Common aerobic pathogens include Staphylococcus aureus, viridans streptococcus, Corynebacterium, Pasteurella multocida, and various gram-negative enteric organisms. The most common anaerobic pathogens include Bacteroides, Fusobacterium, Peptostreptococcus, and Actinomyces species. Pasteurella multocida, an aerobic, facultative anaerobic gram-negative pathogen, is the most commonly isolated pathogen in cat bites and is present in up to 50% of infected wounds. Symptoms of Pasteurella multocida infection develop rapidly within 24 hours and typically include cellulitis and seropurulent drainage. Other uncommon infections have been reported after cat bites, including tularemia (Francisella tularensis) and cat-scratch disease (Bartonella henselae).

Patients with severe wounds (crush injury, joint involvement, deep space infection), cellulitis, sepsis, diabetes, immunosuppression, or other high-risk factors for treatment failure should receive parenteral antibiotics. Alternatives to amoxicillin and clavulanate potassium or ampicillin and sulbactam include other penicillins or cephalosporins, provided they display adequate anaerobic coverage. In the scenario described, the choices containing amoxicillin, ampicillin, and cefazolin are not appropriate because of the patient’s penicillin allergy. Additionally, this patient will require intravenous antibiotics because of the wound severity, so amoxicillin and clavulanate potassium would not be appropriate. The combination of tetracycline and TMP/SMX alone does not provide adequate anaerobic coverage.

178
Q

A 43-year-old healthy man comes to the office because of acute swelling over the dorsum of his dominant right wrist. He says that movement of the wrist is painful. No preceding trauma is noted. Which of the following is the most sensitive test to diagnose septic wrist arthritis in this patient?
A ) Carpal bone radiographic destructive changes
B ) Joint aspirate Gram stain
C ) Serum erythrocyte sedimentation rate
D ) Serum white cell count
E ) Synovial fluid white cell count

A

E ) Synovial fluid white cell count

Analysis of synovial fluid is the most useful diagnostic test. The most definitive test of the joint fluid, bacterial Gram stain and culture, is unfortunately not consistently positive in an acute septic joint. Culture may be negative in about 50% of cases, especially gonococcal, and Gram stain is even less frequently positive. One has to wait a few days for culture results.

As one lowers the threshold for the white cell count in a joint aspirate, the test becomes more sensitive, but less specific. If the white cell count threshold is set at 50,000/mL, which is traditionally the value considered to be the threshold for diagnosis of a septic joint, the sensitivity is 61%. If a white cell count at 17,500 is set as the threshold, the sensitivity is 83%, but specificity is 67%. The specificity rises to 77% if the threshold for the white cell count is raised to 100,000.

Sedimentation rate, serum white cell count, and C-reactive protein do little to change pretest probability of a septic joint.

Radiographic changes are a late finding in septic arthritis.

179
Q
A 25-year-old man comes to the emergency department because of swelling and pain in the finger. He says that he first started noticing symptoms 4 days ago. Physical examination shows warmth of the fingertip and fluctuance of the nail fold. A photograph is shown. Which of the following bacteria is the most likely cause of this infection?
A ) Enterococcus faecalis
B ) Mycobacterium marinum
C ) Pasteurella multocida
D ) Pseudomonas aeruginosa
E ) Staphylococcus aureus
A

E ) Staphylococcus aureus

A paronychia is an infection occurring in the space between the nail plate and the paronychial or eponychial fold. Common causes include nail biting and aggressive manicuring. In an otherwise healthy individual, Staphylococcus aureus and Streptococcus species are the most common pathogens causing a paronychia.

The remaining bacteria listed would only be expected to cause a paronychia in special circumstances. Enterococcus is an enteric bacterium. Mycobacterium marinum can be present in aquatic environments including fish tanks and has also been reported in the Chesapeake Bay. Pasteurella multocida is a common flora of cat mouths. Pseudomonas occurs in areas with moist environments and can be problematic in a burn unit.

180
Q
A 3-year-old boy is brought to the office because of swelling of the right upper and lower eyelids 4 days after sustaining a dog bite to the ipsilateral cheek. Oral temperature is 101.3°F (38.5°C). Examination shows two puncture marks with scant purulent discharge on the right mid cheek, proptosis of the right eye, and swelling of the right eyelids. Which of the following is the most appropriate next step to establish the diagnosis?
A ) Blood cultures
B ) Complete blood count
C ) CT scan
D ) Waters view x-ray study
E ) Wound swab
A

C ) CT scan

Orbital cellulitis may be caused by local extension of a superficial infection such as a hordeolum, or from adjacent infected sinuses, or by hematogenous seeding from a dental or facial abscess. It is important to distinguish between preseptal and orbital cellulitis, which should be suspected when unilateral proptosis is present, and to identify the presence of an intraorbital abscess. Infection within the orbit has implications for management and prognosis due to the effect of high mechanical pressures generated in an unyielding compartment, which will manifest as proptosis, and other symptoms including diplopia, ophthalmoplegia, and chemosis. Orbital cellulitis requires hospitalization and prompt initiation of parenteral antibiotics. An intraorbital abscess can form if treatment is delayed, further compounding the intraorbital pressure and causing tissue necrosis. An intraorbital abscess requires immediate surgical drainage due to risk of blindness. Preseptal cellulitis is an infection of periorbital tissues superficial to the orbital septum, and if the source is from a nearby superficial skin infection, then initial management may be done with oral antibiotics and close follow-up. Preseptal cellulitis from a hematogenous source is more likely to become complicated with orbital cellulitis and abscess if it is not managed with parenteral antibiotics. Infections inside the orbit may extend to the central nervous system and may also cause cavernous sinus thrombosis. CT scan or MRI will confirm intraorbital infection, and related sources such as a facial or dental abscess or a sinusitis, and can direct the surgeon to areas requiring surgical drainage.

Blood cultures are useful in identifying the causative organism of the infection during periods of bacteremia, and are an appropriate test in the general workup of the patient; however, blood cultures alone do not diagnose or exclude intraorbital infection.

Cell counts are also useful in the general workup of the patient to characterize the systemic impact of the infection, and to be used as a guide in the responsiveness to treatment. The counts do not diagnose or exclude intraorbital infection specifically.

On Waters view x-ray studies, increased facial or eyelid edema may be seen as a slight diffuse opacification, but they are unable to delineate orbital and periorbital soft tissues individually. Sinus opacification can be suggestive of active sinusitis and a potential cause of orbital cellulitis. In general, plain x-ray study offers far less information than CT scan or MRI and is not diagnostic of intraorbital infection.

Wound swabs are helpful in identifying the causative organism of the infection when there is an obvious source, and are an appropriate test in the general workup of the patient; however, wound swabs alone do not diagnose, exclude, or characterize the extent of intraorbital infection.